Find a solution: $3(x^2+y^2+z^2)=10(xy+yz+zx)$

As far as I understand - this is the site for solving the problem. Programming and calculation using the computer is not mathematics. If you want to calculate - there is a special section. https://mathematica.stackexchange.com/questions

Here it is necessary to solve the equations.

For the equation:

$$3(x^2+y^2+z^2)=10(xy+xz+yz)$$

The solution is simple.

$$x=4ps$$

$$y=3p^2-10ps+7s^2$$

$$z=p^2-10ps+21s^2$$

$p,s - $ any integer which we ask.

Why make a program? What's the point? For what?


When he wrote the equation he meant probably that entry.

$$q(x^2+y^2+z^2)=(3q+1)(xy+xz+yz)$$

It turns out, this equation has a connection with the Pell equation:

$$p^2-5s^2=\pm1$$

For $+1$ it is necessary to use the first solution $(9 ; 4)$. For $-1$ it is necessary to use the first solution $(2 ; 1)$. Knowing what the decision can be found on the following formula.

$$p_2=9p_1+20s_1$$

$$s_2=4p_1+9s_1$$

Using the solutions of the Pell equation can be found when there are solutions. $q=\mp(p^2-s^2)$

Will make a replacement. $t=\mp4ps$ Then the solution can be written:

$$x=2(q+1)tkn$$

$$y=(q+t+1)k^2-2(3q+1)tkn+(t-q-1)(10q^2+7q+1)n^2$$

$$z=(t-q-1)k^2-2(3q+1)tkn+(t+q+1)(10q^2+7q+1)n^2$$

$k,n $ - integers asked us. May be necessary, after all the calculations is to obtain a relatively simple solution, divided by the common divisor.


This was a bunch of nonsense characters typed by hand so that the software would not test me with a ``captcha''

   0           1           3
   0           3           1
   1           0           3
   1           3           0
   3           0           1
   3           1           0
   3           9          40
   3          40           9
   5          32         119
   5         119          32
   8          11          65
   8          65          11
   9           3          40
   9          40           3
  11           8          65
  11          65           8
  13          15          96
  13          96          15
  15          13          96
  15          96          13
  32           5         119
  32         119           5
  40           3           9
  40           9           3
  65           8          11
  65          11           8
  96          13          15
  96          15          13
 119           5          32
 119          32           5

Because the equation is homogenous, the integer solutions can be derived from the rational solutions, in other words swapping between projective and affine form.

I prove below that the set of non-zero rational solutions are common rational multiples of the following (which, conversely, satisfies the equation identically) for any rational parameter t:

$x,\ y,\ z = 2 t - 1,\ 3 t^2 - 8 t + 5,\ t^2 - 6 t + 8$

So, explicitly, the complete set of integer solutions with GCD(x, y, z) = 1 can be expressed as follows, as $m,\ n$ range over coprime integer pairs

$x,\ y,\ z = (2 m - n) n,\ 3 m^2 - 8 m n + 5 n^2,\ m^2 - 6 m n + 8 n^2$

Proof:

Let $p,\ q,\ r = - x + y + z,\ x - y + z,\ x + y - z$

<=> $2 x,\ 2 y,\ 2 z = q + r,\ r + p,\ p + q$

Then $p^2 + q^2 + r^2 = 3 (x^2 + y^2 + z^2) - 2 (x y + y z + z x)$

and $p q + q r + r p = - (x^2 + y^2 + z^2) + 2 (x y + y z + z x)$

So if $a (p^2 + q^2 + r^2) = b (p q + q r + r p)$

then $(3 a + b) (x^2 + y^2 + z^2) = 2 (a + b) (x y + y z + z x)$

So the required equation is obtained with $3 a + b,\ a + b = 3,\ 5$, i.e. $a,\ b = -1,\ 6$ and the original is equivalent to

$p^2 + q^2 + r^2 + 6 (p q + q r + r p) = 0$

If r = 0 then this becomes $(p + 3 q)^2 = 8 q^2$, which for rational $p, q$ has only the solution p = q = 0

Otherwise, we can replace $\frac{p}{r},\ \frac{q}{r}$ by $p,\ q$ respectively and the equation becomes

$q^2 + 6 (p + 1) q + (p^2 + 6 p + 1) = 0$

which for rational $q$ (assuming rational $p$) requires rational $s$ with

$q = 2 s - 3 (p + 1)$

$9 (p + 1)^2 - (p^2 + 6 p + 1) = 4 s^2$

The latter is equivalent to $8 s^2 - (4 p + 3)^2 = 7$

So in view of the obvious rational solution $4 p + 3,\ s = 1, 1$ we can replace in this $4 p + 3,\ s = 2 t u + 1,\ u + 1$

which implies either $u = 0$, which recovers the solution already observed, or $u = \frac{4 - t}{t^2 - 2}$

which gives successively

$p = \frac{- t^2 + 2 t + 1}{t^2 - 2}$

$s = \frac{t^2 - t + 2}{t^2 - 2}$

$q = \frac{2 v^2 - 8 v + 7}{t^2 - 2}$

So the original $p,\ q,\ r$ can be taken as follows, and the result follows

$p,\ q,\ r = - t^2 + 2 t + 1,\ 2 t^2 - 8 t + 7,\ t^2 - 2$

Sanity check: In the expressions for $x, y, z$ take $t = 0$ to give $x, y, z = 1, 5, 8$ and the equation becomes 2.3^3.5 = 2.3^3.5

Regards

John R Ramsden


august 2020, this is the good answer, gradually deleting my others here, limit of 5 per day

ADDED: Another way of saying this: Given integers $B > A > 0,$ with $\gcd(A,B) = 1,$ then there is a solution in integers $x,y,z,$ not all $0,$ to $$ A(x^2 + y^2 + z^2) - B (yz + zx + xy) = 0, $$ if and only if both $B+2A$ and $B-A$ are integrally represented by the binary form $u^2 + 3 v^2.$

ORIGINAL: It is simpler than I had feared. We take $0 < A < B,$ and $\gcd(A,B)=1.$ After that, what we are really concerned about are the two numbers that come up in diagonalizing the form, those being $B + 2A$ and $B-A.$

The form is isotropic over the rationals (and integers) if and only if:

(I) when factoring both $B + 2A$ and $B-A,$ the exponents of $2$ are even.

(II) when factoring both $B + 2A$ and $B-A,$ the exponents of $q$ are even, where $q \equiv 5 \pmod 6$ is a prime.

That's it. Note that we could combine these into one test, for all primes $p \equiv 2 \pmod 3.$

Here is an example, $$ 6(x^2 + y^2 + z^2) = 55 (yz+zx+xy) $$ All primitive solutions can be found by ordering the elements of three Pythagorean Triple type recipes for $(x,y,z).$ The theorem is that a finite number of such recipes succeed; for this problem the count required has turned out to be one of $1,2,3,4,6,8,12,16.$ That is, either $2^k$ or $3 \cdot 2^k$ The quadratic form is so symmetric that each matrix of coefficients comes out in an amusing cyclic pattern.

$$ x= 48 u^2 + 97uv + 34v^2 \; , \; \; y= 34 u^2 -29uv -15v^2 \; , \; \; z = -15 u^2 -uv + 48 v^2 $$

$$ x= 54 u^2 + 91uv + 25v^2 \; , \; \; y= 25 u^2 -41uv -12v^2 \; , \; \; z = -12 u^2 +17uv + 54 v^2 $$

$$ x= 60 u^2 + 71uv + 9v^2 \; , \; \; y= 9 u^2 -53uv -2v^2 \; , \; \; z = -2 u^2 +49uv + 60 v^2 $$

Here are the three recipes sorted and with the $u,v$ values specified.

august 2020 my good answer

=-=-=-=-=-=-=-=-=-=-=-=-=-=-=-=-=-=-=-=
         x         y         z  such that x >= |y| >= |z| 
        48        34       -15
        48        34       -15      < 48, 97, 34 >      1  0    
        54        25       -12
        54        25       -12      < 54, 91, 25 >      1  0    
        60         9        -2
        60         9        -2      < 60, 71, 9 >      1  0    
       140       107       -46
       140       107       -46      < 60, 71, 9 >      1  1    
       170        59       -28
       170        59       -28      < 54, 91, 25 >      1  1    
       179        32       -10
       179        32       -10      < 48, 97, 34 >      1  1    
       391       150       -72
       391       150       -72      < 60, 71, 9 >      2  1    
       423        40         6
       423        40         6      < 54, 91, 25 >      2  1     POSITIVE 
       552       525      -206
       552       525      -206      < 54, 91, 25 >      1  3    
       645       414      -188
       645       414      -188      < 48, 97, 34 >      1  3    
       685       354      -168
       685       354      -168      < 60, 71, 9 >      1  3    
       757       204       -90
       757       204       -90      < 48, 97, 34 >      3  1    
       762       189       -80
       762       189       -80      < 60, 71, 9 >      3  1    
       784        90        -3
       784        90        -3      < 54, 91, 25 >      3  1    
       826       747      -300
       826       747      -300      < 60, 71, 9 >      2  3    
       920       818      -331
       920       818      -331      < 54, 91, 25 >      1  4    
       987       540      -254
       987       540      -254      < 54, 91, 25 >      2  3    
      1002       516      -245
      1002       516      -245      < 60, 71, 9 >      3  2    
=-=-=-=-=-=-=-=-=-=-=-=-=-=-=-=-=-=-=-=
         x         y         z  such that x >= |y| >= |z|

august 2020 my good answer